Add solutions for Serway and Jewett v8's chapter 31.
[course.git] / latex / problems / Serway_and_Jewett_8 / problem31.31.tex
1 \begin{problem*}{31.31}
2 Two parallel rails with negligable resistance are $10.0\U{cm}$ apart
3 and are connected by a resistor of resistance $R_3=5.00\U{\Ohm}$.  The
4 circuit also contains two metal rods having resistances of
5 $R_1=10.0\U{\Ohm}$ and $R_2=15.0\U{\Ohm}$ sliding along the rails
6 (Fig.~P31.31).  The rods are pulled away from the resistor at constant
7 speeds of $v_1=4.00\U{m/s}$ and $v_2=2.00\U{m/s}$, respectively.  A
8 uniform magnetic field of magnitude $B=0.0100\U{T}$ is applied
9 perpendicular to the plane of the rails.  Determine the current in
10 $R_3$.
11 \begin{center}
12 % --+-----+-----+--
13 % x | x x Z x x | x
14 % <-| x x ZR3 x |->
15 % v1| x x Z x x |v2
16 % --+-----+-----+--
17 %   R1            R2
18 \begin{asy}
19 import Mechanics;
20 import ElectroMag;
21 import Circ;
22
23 real u = 1cm;
24 real w = 1mm;
25
26 MultiTerminal R3 = resistor(dir=90, "$R_3$", draw=false);
27 real rlen = R3.terminal[1].y - R3.terminal[0].y;
28 Vector B = BField(phi=-90);
29 vector_field(R3.center, width=3*u, height=rlen, v=B);
30 R3.draw();
31 real yt = R3.terminal[1].y;
32 real yb = R3.terminal[0].y;
33 wire((-1.5*u, yt), (1.5*u, yt));
34 wire((-1.5*u, yb), (1.5*u, yb));
35 dot(R3.terminal[0]);
36 dot(R3.terminal[1]);
37 pair p1 = (-u,(yt+yb)/2);
38 Vector v1 = Velocity(p1, dir=180, "$v_1$");  v1.draw();
39 Block R1 = Block(p1, width=w, height=rlen);  R1.draw();
40 label("$R_1$", (p1.x, yb), align=S);
41 pair p2 = (-p1.x, p1.y);
42 Vector v1 = Velocity(p2, "$v_2$");  v1.draw();
43 Block R2 = Block(p2, width=w, height=rlen);  R2.draw();
44 label("$R_2$", (p2.x, yb), align=S);
45 \end{asy}
46 \end{center}
47 \end{problem*}
48
49 \begin{solution}
50 The magnetic flux through the loop will increase as the area enclosed
51 increases, inducing \EMF\ in each rod.
52 \begin{align}
53   \EMF_1 &= BLv_1 = 4.00\U{mV} \\
54   \EMF_2 &= BLv_2 = 2.00\U{mV} \;.
55 \end{align}
56 The direction of the induced \EMF{}s will try to resist the increasing
57 flux, so $\EMF_1$ will be directed downward and $\EMF_2$ will be
58 directed upward.  The situation is then equivalent to the following
59 circuit:
60 \begin{center}
61 \begin{asy}
62 import Circ;
63
64 real u = 1cm;
65
66 MultiTerminal V1 = source(dir=-90, type=DC, Label("$\EMF_1$", align=W));
67 MultiTerminal R1 = resistor(V1.terminal[1], dir=-90, Label("$R_1$", align=W));
68 MultiTerminal R3 = resistor(dir=90, "$R_3$", draw=false);
69 R3.centerto(V1.terminal[0], R1.terminal[1], offset=u);  R3.draw();
70 MultiTerminal V2 = source(R1.terminal[1] + (2*u, 0), dir=90, type=DC,
71     Label("$\EMF_2$", align=E));
72 MultiTerminal R2 = resistor(V2.terminal[1], dir=90, Label("$R_2$", align=E));
73 wire(V1.terminal[0], R2.terminal[1]);
74 wire(R1.terminal[1], V2.terminal[0]);
75 pair top = (R3.center.x, V1.terminal[0].y);
76 pair bot = (top.x, R1.terminal[1].y);
77 wire(top, R3.terminal[0]);
78 wire(bot, R3.terminal[1]);
79 dot(top);
80 dot(bot);
81 \end{asy}
82 \end{center}
83 Solve this circuit using the usual Kirchhoff approach.  Label the
84 current through the three branches $I_1$, $I_2$, and $I_3$ each
85 pointing up, and you have
86 \begin{align}
87   I_1 + I_2 + I_3 &= 0 \\
88   \EMF_1 + I_1 R_1 - I_3 R_3 &= 0 \\
89   \EMF_2 - I_2 R_2 + I_3 R_3 &= 0 \;.
90 \end{align}
91 This gives three equations with three unknowns.  Solve using your
92 favorite method.
93 \begin{align}
94  \begin{pmatrix}
95   0 \\
96   \EMF_1 \\
97   \EMF_2
98  \end{pmatrix}
99   &=
100  \begin{pmatrix}
101   1\U{\Ohm} & 1\U{\Ohm} & 1\U{\Ohm} \\
102   -R_1 & 0 & R_3 \\
103   0 & R_2 & -R_3
104  \end{pmatrix}
105  \begin{pmatrix}
106   I_1 \\
107   I_2 \\
108   I_3
109  \end{pmatrix} \\
110  \begin{pmatrix}
111   I_1 \\
112   I_2 \\
113   I_3
114  \end{pmatrix}
115   =&
116  \begin{pmatrix}
117   1\U{\Ohm} & 1\U{\Ohm} & 1\U{\Ohm} \\
118   -R_1 & 0 & R_3 \\
119   0 & R_2 & -R_3
120  \end{pmatrix}^{-1}
121  \begin{pmatrix}
122   0 \\
123   \EMF_1 \\
124   \EMF_2
125  \end{pmatrix}
126   =
127  \begin{pmatrix}
128   -327 \\
129   182 \\
130   145
131  \end{pmatrix} \U{$\mu$A}
132  \;.
133 \end{align}
134 Therefore, \ans{$I_3=145\U{$\mu$A}$ upwards}.
135 \end{solution}